New Order #5: where Fibonacci and Beatty meet at Wythoff












6












$begingroup$


Introduction (may be ignored)



Putting all positive numbers in its regular order (1, 2, 3, ...) is a bit boring, isn't it? So here is a series of challenges around permutations (reshuffelings) of all positive numbers. This is the fifth challenge in this series (links to the first, second, third and fourth challenge).



In this challenge, we will meet the Wythoff array, which is a intertwined avalanche of Fibonacci sequences and Beatty sequences!



The Fibonacci numbers are probably for most of you a well known sequence. Given two starting numbers $F_0$ and $F_1$, the following $F_n$ are given by: $F_n = F_{(n-1)} + F_{(n-2)}$ for $n>2$.



The Beatty sequence, given a parameter $r$ is: $B^r_n = lfloor rn rfloor$ for $n ge 1$. One of the properties of the Beatty sequence is that for every parameter $r$, there is exactly one parameter $s=r/(r-1)$, such that the Beatty sequences for those parameters are disjunct and joined together, they span all natural numbers excluding 0 (e.g.: $B^r cup B^{r/(r-1)} = Bbb{N} setminus {0}$).



Now here comes the mindblowing part: you can create an array, where each row is a Fibonacci sequence and each column is a Beatty sequence. This array is the Wythoff array. The best part is: every positive number appears exactly once in this array! The array looks like this:



   1    2    3    5    8   13   21   34   55   89  144 ...
4 7 11 18 29 47 76 123 199 322 521 ...
6 10 16 26 42 68 110 178 288 466 754 ...
9 15 24 39 63 102 165 267 432 699 1131 ...
12 20 32 52 84 136 220 356 576 932 1508 ...
14 23 37 60 97 157 254 411 665 1076 1741 ...
17 28 45 73 118 191 309 500 809 1309 2118 ...
19 31 50 81 131 212 343 555 898 1453 2351 ...
22 36 58 94 152 246 398 644 1042 1686 2728 ...
25 41 66 107 173 280 453 733 1186 1919 3105 ...
27 44 71 115 186 301 487 788 1275 2063 3338 ...
...


An element at row $m$ and column $n$ is defined as:



$A_{m,n} = begin{cases}
A_{m,1} = leftlfloor lfloor mvarphi rfloor varphi rightrfloor\
A_{m,2} = leftlfloor lfloor mvarphi rfloor varphi^2 rightrfloor\
A_{m,n} = A_{m,n-2}+A_{m,n-1} text{ for }n > 2
end{cases}$



where $varphi$ is the golden ratio: $varphi=frac{1+sqrt{5}}{2}$.



If we follow the anti-diagonals of this array, we get A035513, which is the target sequence for this challenge (note that this sequence is added to the OEIS by Neil Sloane himself!). Since this is a "pure sequence" challenge, the task is to output $a(n)$ for a given $n$ as input, where $a(n)$ is A035513.



There are different strategies you can follow to get to $a(n)$, which makes this challenge (in my opinion) really interesting.



Task



Given an integer input $n$, output $a(n)$ in integer format, where $a(n)$ is A035513.



Note: 1-based indexing is assumed here; you may use 0-based indexing, so $a(0) = 1; a(1) = 2$, etc. Please mention this in your answer if you choose to use this.



Test cases



Input | Output
---------------
1 | 1
5 | 7
20 | 20
50 | 136
78 | 30
123 | 3194
1234 | 8212236486
3000 | 814
9999 | 740496902
29890 | 637


Rules




  • Input and output are integers (your program should at least support input and output in the range of 1 up to 32767). Note that $a(n)$ goes up to 30 digit numbers in this range...

  • Invalid input (0, floats, strings, negative values, etc.) may lead to unpredicted output, errors or (un)defined behaviour.

  • Default I/O rules apply.


  • Default loopholes are forbidden.

  • This is code-golf, so the shortest answers in bytes wins










share|improve this question









$endgroup$












  • $begingroup$
    So what's the New Order reference here?
    $endgroup$
    – Luis Mendo
    2 hours ago










  • $begingroup$
    @LuisMendo: the avalanche of Fibonacci and Beatty sequences, which form the Wythoff array...
    $endgroup$
    – agtoever
    2 hours ago










  • $begingroup$
    Ah, I completely missed that! Now I feel regret...
    $endgroup$
    – Luis Mendo
    2 hours ago










  • $begingroup$
    Is a floating point representation of phi (or rt(5)) and application of the recurrence going to satisfy the range requirement?
    $endgroup$
    – Jonathan Allan
    2 hours ago










  • $begingroup$
    @JonathanAllan : good point... I'll look into that later. For now: let's pose that if some code passes the test cases, then it works sufficiently.
    $endgroup$
    – agtoever
    2 hours ago


















6












$begingroup$


Introduction (may be ignored)



Putting all positive numbers in its regular order (1, 2, 3, ...) is a bit boring, isn't it? So here is a series of challenges around permutations (reshuffelings) of all positive numbers. This is the fifth challenge in this series (links to the first, second, third and fourth challenge).



In this challenge, we will meet the Wythoff array, which is a intertwined avalanche of Fibonacci sequences and Beatty sequences!



The Fibonacci numbers are probably for most of you a well known sequence. Given two starting numbers $F_0$ and $F_1$, the following $F_n$ are given by: $F_n = F_{(n-1)} + F_{(n-2)}$ for $n>2$.



The Beatty sequence, given a parameter $r$ is: $B^r_n = lfloor rn rfloor$ for $n ge 1$. One of the properties of the Beatty sequence is that for every parameter $r$, there is exactly one parameter $s=r/(r-1)$, such that the Beatty sequences for those parameters are disjunct and joined together, they span all natural numbers excluding 0 (e.g.: $B^r cup B^{r/(r-1)} = Bbb{N} setminus {0}$).



Now here comes the mindblowing part: you can create an array, where each row is a Fibonacci sequence and each column is a Beatty sequence. This array is the Wythoff array. The best part is: every positive number appears exactly once in this array! The array looks like this:



   1    2    3    5    8   13   21   34   55   89  144 ...
4 7 11 18 29 47 76 123 199 322 521 ...
6 10 16 26 42 68 110 178 288 466 754 ...
9 15 24 39 63 102 165 267 432 699 1131 ...
12 20 32 52 84 136 220 356 576 932 1508 ...
14 23 37 60 97 157 254 411 665 1076 1741 ...
17 28 45 73 118 191 309 500 809 1309 2118 ...
19 31 50 81 131 212 343 555 898 1453 2351 ...
22 36 58 94 152 246 398 644 1042 1686 2728 ...
25 41 66 107 173 280 453 733 1186 1919 3105 ...
27 44 71 115 186 301 487 788 1275 2063 3338 ...
...


An element at row $m$ and column $n$ is defined as:



$A_{m,n} = begin{cases}
A_{m,1} = leftlfloor lfloor mvarphi rfloor varphi rightrfloor\
A_{m,2} = leftlfloor lfloor mvarphi rfloor varphi^2 rightrfloor\
A_{m,n} = A_{m,n-2}+A_{m,n-1} text{ for }n > 2
end{cases}$



where $varphi$ is the golden ratio: $varphi=frac{1+sqrt{5}}{2}$.



If we follow the anti-diagonals of this array, we get A035513, which is the target sequence for this challenge (note that this sequence is added to the OEIS by Neil Sloane himself!). Since this is a "pure sequence" challenge, the task is to output $a(n)$ for a given $n$ as input, where $a(n)$ is A035513.



There are different strategies you can follow to get to $a(n)$, which makes this challenge (in my opinion) really interesting.



Task



Given an integer input $n$, output $a(n)$ in integer format, where $a(n)$ is A035513.



Note: 1-based indexing is assumed here; you may use 0-based indexing, so $a(0) = 1; a(1) = 2$, etc. Please mention this in your answer if you choose to use this.



Test cases



Input | Output
---------------
1 | 1
5 | 7
20 | 20
50 | 136
78 | 30
123 | 3194
1234 | 8212236486
3000 | 814
9999 | 740496902
29890 | 637


Rules




  • Input and output are integers (your program should at least support input and output in the range of 1 up to 32767). Note that $a(n)$ goes up to 30 digit numbers in this range...

  • Invalid input (0, floats, strings, negative values, etc.) may lead to unpredicted output, errors or (un)defined behaviour.

  • Default I/O rules apply.


  • Default loopholes are forbidden.

  • This is code-golf, so the shortest answers in bytes wins










share|improve this question









$endgroup$












  • $begingroup$
    So what's the New Order reference here?
    $endgroup$
    – Luis Mendo
    2 hours ago










  • $begingroup$
    @LuisMendo: the avalanche of Fibonacci and Beatty sequences, which form the Wythoff array...
    $endgroup$
    – agtoever
    2 hours ago










  • $begingroup$
    Ah, I completely missed that! Now I feel regret...
    $endgroup$
    – Luis Mendo
    2 hours ago










  • $begingroup$
    Is a floating point representation of phi (or rt(5)) and application of the recurrence going to satisfy the range requirement?
    $endgroup$
    – Jonathan Allan
    2 hours ago










  • $begingroup$
    @JonathanAllan : good point... I'll look into that later. For now: let's pose that if some code passes the test cases, then it works sufficiently.
    $endgroup$
    – agtoever
    2 hours ago
















6












6








6





$begingroup$


Introduction (may be ignored)



Putting all positive numbers in its regular order (1, 2, 3, ...) is a bit boring, isn't it? So here is a series of challenges around permutations (reshuffelings) of all positive numbers. This is the fifth challenge in this series (links to the first, second, third and fourth challenge).



In this challenge, we will meet the Wythoff array, which is a intertwined avalanche of Fibonacci sequences and Beatty sequences!



The Fibonacci numbers are probably for most of you a well known sequence. Given two starting numbers $F_0$ and $F_1$, the following $F_n$ are given by: $F_n = F_{(n-1)} + F_{(n-2)}$ for $n>2$.



The Beatty sequence, given a parameter $r$ is: $B^r_n = lfloor rn rfloor$ for $n ge 1$. One of the properties of the Beatty sequence is that for every parameter $r$, there is exactly one parameter $s=r/(r-1)$, such that the Beatty sequences for those parameters are disjunct and joined together, they span all natural numbers excluding 0 (e.g.: $B^r cup B^{r/(r-1)} = Bbb{N} setminus {0}$).



Now here comes the mindblowing part: you can create an array, where each row is a Fibonacci sequence and each column is a Beatty sequence. This array is the Wythoff array. The best part is: every positive number appears exactly once in this array! The array looks like this:



   1    2    3    5    8   13   21   34   55   89  144 ...
4 7 11 18 29 47 76 123 199 322 521 ...
6 10 16 26 42 68 110 178 288 466 754 ...
9 15 24 39 63 102 165 267 432 699 1131 ...
12 20 32 52 84 136 220 356 576 932 1508 ...
14 23 37 60 97 157 254 411 665 1076 1741 ...
17 28 45 73 118 191 309 500 809 1309 2118 ...
19 31 50 81 131 212 343 555 898 1453 2351 ...
22 36 58 94 152 246 398 644 1042 1686 2728 ...
25 41 66 107 173 280 453 733 1186 1919 3105 ...
27 44 71 115 186 301 487 788 1275 2063 3338 ...
...


An element at row $m$ and column $n$ is defined as:



$A_{m,n} = begin{cases}
A_{m,1} = leftlfloor lfloor mvarphi rfloor varphi rightrfloor\
A_{m,2} = leftlfloor lfloor mvarphi rfloor varphi^2 rightrfloor\
A_{m,n} = A_{m,n-2}+A_{m,n-1} text{ for }n > 2
end{cases}$



where $varphi$ is the golden ratio: $varphi=frac{1+sqrt{5}}{2}$.



If we follow the anti-diagonals of this array, we get A035513, which is the target sequence for this challenge (note that this sequence is added to the OEIS by Neil Sloane himself!). Since this is a "pure sequence" challenge, the task is to output $a(n)$ for a given $n$ as input, where $a(n)$ is A035513.



There are different strategies you can follow to get to $a(n)$, which makes this challenge (in my opinion) really interesting.



Task



Given an integer input $n$, output $a(n)$ in integer format, where $a(n)$ is A035513.



Note: 1-based indexing is assumed here; you may use 0-based indexing, so $a(0) = 1; a(1) = 2$, etc. Please mention this in your answer if you choose to use this.



Test cases



Input | Output
---------------
1 | 1
5 | 7
20 | 20
50 | 136
78 | 30
123 | 3194
1234 | 8212236486
3000 | 814
9999 | 740496902
29890 | 637


Rules




  • Input and output are integers (your program should at least support input and output in the range of 1 up to 32767). Note that $a(n)$ goes up to 30 digit numbers in this range...

  • Invalid input (0, floats, strings, negative values, etc.) may lead to unpredicted output, errors or (un)defined behaviour.

  • Default I/O rules apply.


  • Default loopholes are forbidden.

  • This is code-golf, so the shortest answers in bytes wins










share|improve this question









$endgroup$




Introduction (may be ignored)



Putting all positive numbers in its regular order (1, 2, 3, ...) is a bit boring, isn't it? So here is a series of challenges around permutations (reshuffelings) of all positive numbers. This is the fifth challenge in this series (links to the first, second, third and fourth challenge).



In this challenge, we will meet the Wythoff array, which is a intertwined avalanche of Fibonacci sequences and Beatty sequences!



The Fibonacci numbers are probably for most of you a well known sequence. Given two starting numbers $F_0$ and $F_1$, the following $F_n$ are given by: $F_n = F_{(n-1)} + F_{(n-2)}$ for $n>2$.



The Beatty sequence, given a parameter $r$ is: $B^r_n = lfloor rn rfloor$ for $n ge 1$. One of the properties of the Beatty sequence is that for every parameter $r$, there is exactly one parameter $s=r/(r-1)$, such that the Beatty sequences for those parameters are disjunct and joined together, they span all natural numbers excluding 0 (e.g.: $B^r cup B^{r/(r-1)} = Bbb{N} setminus {0}$).



Now here comes the mindblowing part: you can create an array, where each row is a Fibonacci sequence and each column is a Beatty sequence. This array is the Wythoff array. The best part is: every positive number appears exactly once in this array! The array looks like this:



   1    2    3    5    8   13   21   34   55   89  144 ...
4 7 11 18 29 47 76 123 199 322 521 ...
6 10 16 26 42 68 110 178 288 466 754 ...
9 15 24 39 63 102 165 267 432 699 1131 ...
12 20 32 52 84 136 220 356 576 932 1508 ...
14 23 37 60 97 157 254 411 665 1076 1741 ...
17 28 45 73 118 191 309 500 809 1309 2118 ...
19 31 50 81 131 212 343 555 898 1453 2351 ...
22 36 58 94 152 246 398 644 1042 1686 2728 ...
25 41 66 107 173 280 453 733 1186 1919 3105 ...
27 44 71 115 186 301 487 788 1275 2063 3338 ...
...


An element at row $m$ and column $n$ is defined as:



$A_{m,n} = begin{cases}
A_{m,1} = leftlfloor lfloor mvarphi rfloor varphi rightrfloor\
A_{m,2} = leftlfloor lfloor mvarphi rfloor varphi^2 rightrfloor\
A_{m,n} = A_{m,n-2}+A_{m,n-1} text{ for }n > 2
end{cases}$



where $varphi$ is the golden ratio: $varphi=frac{1+sqrt{5}}{2}$.



If we follow the anti-diagonals of this array, we get A035513, which is the target sequence for this challenge (note that this sequence is added to the OEIS by Neil Sloane himself!). Since this is a "pure sequence" challenge, the task is to output $a(n)$ for a given $n$ as input, where $a(n)$ is A035513.



There are different strategies you can follow to get to $a(n)$, which makes this challenge (in my opinion) really interesting.



Task



Given an integer input $n$, output $a(n)$ in integer format, where $a(n)$ is A035513.



Note: 1-based indexing is assumed here; you may use 0-based indexing, so $a(0) = 1; a(1) = 2$, etc. Please mention this in your answer if you choose to use this.



Test cases



Input | Output
---------------
1 | 1
5 | 7
20 | 20
50 | 136
78 | 30
123 | 3194
1234 | 8212236486
3000 | 814
9999 | 740496902
29890 | 637


Rules




  • Input and output are integers (your program should at least support input and output in the range of 1 up to 32767). Note that $a(n)$ goes up to 30 digit numbers in this range...

  • Invalid input (0, floats, strings, negative values, etc.) may lead to unpredicted output, errors or (un)defined behaviour.

  • Default I/O rules apply.


  • Default loopholes are forbidden.

  • This is code-golf, so the shortest answers in bytes wins







code-golf sequence






share|improve this question













share|improve this question











share|improve this question




share|improve this question










asked 2 hours ago









agtoeveragtoever

1,359424




1,359424












  • $begingroup$
    So what's the New Order reference here?
    $endgroup$
    – Luis Mendo
    2 hours ago










  • $begingroup$
    @LuisMendo: the avalanche of Fibonacci and Beatty sequences, which form the Wythoff array...
    $endgroup$
    – agtoever
    2 hours ago










  • $begingroup$
    Ah, I completely missed that! Now I feel regret...
    $endgroup$
    – Luis Mendo
    2 hours ago










  • $begingroup$
    Is a floating point representation of phi (or rt(5)) and application of the recurrence going to satisfy the range requirement?
    $endgroup$
    – Jonathan Allan
    2 hours ago










  • $begingroup$
    @JonathanAllan : good point... I'll look into that later. For now: let's pose that if some code passes the test cases, then it works sufficiently.
    $endgroup$
    – agtoever
    2 hours ago




















  • $begingroup$
    So what's the New Order reference here?
    $endgroup$
    – Luis Mendo
    2 hours ago










  • $begingroup$
    @LuisMendo: the avalanche of Fibonacci and Beatty sequences, which form the Wythoff array...
    $endgroup$
    – agtoever
    2 hours ago










  • $begingroup$
    Ah, I completely missed that! Now I feel regret...
    $endgroup$
    – Luis Mendo
    2 hours ago










  • $begingroup$
    Is a floating point representation of phi (or rt(5)) and application of the recurrence going to satisfy the range requirement?
    $endgroup$
    – Jonathan Allan
    2 hours ago










  • $begingroup$
    @JonathanAllan : good point... I'll look into that later. For now: let's pose that if some code passes the test cases, then it works sufficiently.
    $endgroup$
    – agtoever
    2 hours ago


















$begingroup$
So what's the New Order reference here?
$endgroup$
– Luis Mendo
2 hours ago




$begingroup$
So what's the New Order reference here?
$endgroup$
– Luis Mendo
2 hours ago












$begingroup$
@LuisMendo: the avalanche of Fibonacci and Beatty sequences, which form the Wythoff array...
$endgroup$
– agtoever
2 hours ago




$begingroup$
@LuisMendo: the avalanche of Fibonacci and Beatty sequences, which form the Wythoff array...
$endgroup$
– agtoever
2 hours ago












$begingroup$
Ah, I completely missed that! Now I feel regret...
$endgroup$
– Luis Mendo
2 hours ago




$begingroup$
Ah, I completely missed that! Now I feel regret...
$endgroup$
– Luis Mendo
2 hours ago












$begingroup$
Is a floating point representation of phi (or rt(5)) and application of the recurrence going to satisfy the range requirement?
$endgroup$
– Jonathan Allan
2 hours ago




$begingroup$
Is a floating point representation of phi (or rt(5)) and application of the recurrence going to satisfy the range requirement?
$endgroup$
– Jonathan Allan
2 hours ago












$begingroup$
@JonathanAllan : good point... I'll look into that later. For now: let's pose that if some code passes the test cases, then it works sufficiently.
$endgroup$
– agtoever
2 hours ago






$begingroup$
@JonathanAllan : good point... I'll look into that later. For now: let's pose that if some code passes the test cases, then it works sufficiently.
$endgroup$
– agtoever
2 hours ago












3 Answers
3






active

oldest

votes


















1












$begingroup$


Jelly, 30 bytes



p`SÞ⁸ịð;Øp,²;¤×Ḟ¥/;+ƝQƊ⁹¡ị@ð/


Try it online!

This is a little slow, but a huge improvement is made with a prefix of Ḥ½Ċ (double, square-root, ceiling) like in this test-suite.






share|improve this answer











$endgroup$









  • 1




    $begingroup$
    you are right! 740496902 is the result for 999
    $endgroup$
    – J42161217
    47 mins ago










  • $begingroup$
    Combining the first part of yours and second part of mine gives 25 bytes. Not sure which of us should have the combined version!
    $endgroup$
    – Nick Kennedy
    37 mins ago










  • $begingroup$
    @NickKennedy - nice, go for it!
    $endgroup$
    – Jonathan Allan
    35 mins ago



















1












$begingroup$


Jelly, 27 24 bytes



p`SÞ⁸ịð’;×ØpḞ¥×⁹r‘ÆḞ¤Sð/


Try it online!



Monadic link using 1-based indexing.
Thanks to @JonathanAllan for a better way of getting the row and columns from n and saving 3 bytes. In its shortest form it’s too slow for larger n on TIO, so the following Try it online! reduces the size of the initial list of rows and columns at the cost of three bytes.






share|improve this answer











$endgroup$













  • $begingroup$
    ...×⁹r‘ÆḞ¤Sð/ saves one in your amalgamation version (TIO)
    $endgroup$
    – Jonathan Allan
    9 mins ago





















0












$begingroup$


Wolfram Language (Mathematica), 90 bytes



Flatten[Table[(F=Fibonacci)[a+1]⌊(b-a+1)GoldenRatio⌋+(b-a)F@a,{b,#},{a,b,1,-1}]][[#]]&


Try it online!






share|improve this answer









$endgroup$














    Your Answer






    StackExchange.ifUsing("editor", function () {
    StackExchange.using("externalEditor", function () {
    StackExchange.using("snippets", function () {
    StackExchange.snippets.init();
    });
    });
    }, "code-snippets");

    StackExchange.ready(function() {
    var channelOptions = {
    tags: "".split(" "),
    id: "200"
    };
    initTagRenderer("".split(" "), "".split(" "), channelOptions);

    StackExchange.using("externalEditor", function() {
    // Have to fire editor after snippets, if snippets enabled
    if (StackExchange.settings.snippets.snippetsEnabled) {
    StackExchange.using("snippets", function() {
    createEditor();
    });
    }
    else {
    createEditor();
    }
    });

    function createEditor() {
    StackExchange.prepareEditor({
    heartbeatType: 'answer',
    autoActivateHeartbeat: false,
    convertImagesToLinks: false,
    noModals: true,
    showLowRepImageUploadWarning: true,
    reputationToPostImages: null,
    bindNavPrevention: true,
    postfix: "",
    imageUploader: {
    brandingHtml: "Powered by u003ca class="icon-imgur-white" href="https://imgur.com/"u003eu003c/au003e",
    contentPolicyHtml: "User contributions licensed under u003ca href="https://creativecommons.org/licenses/by-sa/3.0/"u003ecc by-sa 3.0 with attribution requiredu003c/au003e u003ca href="https://stackoverflow.com/legal/content-policy"u003e(content policy)u003c/au003e",
    allowUrls: true
    },
    onDemand: true,
    discardSelector: ".discard-answer"
    ,immediatelyShowMarkdownHelp:true
    });


    }
    });














    draft saved

    draft discarded


















    StackExchange.ready(
    function () {
    StackExchange.openid.initPostLogin('.new-post-login', 'https%3a%2f%2fcodegolf.stackexchange.com%2fquestions%2f183186%2fnew-order-5-where-fibonacci-and-beatty-meet-at-wythoff%23new-answer', 'question_page');
    }
    );

    Post as a guest















    Required, but never shown

























    3 Answers
    3






    active

    oldest

    votes








    3 Answers
    3






    active

    oldest

    votes









    active

    oldest

    votes






    active

    oldest

    votes









    1












    $begingroup$


    Jelly, 30 bytes



    p`SÞ⁸ịð;Øp,²;¤×Ḟ¥/;+ƝQƊ⁹¡ị@ð/


    Try it online!

    This is a little slow, but a huge improvement is made with a prefix of Ḥ½Ċ (double, square-root, ceiling) like in this test-suite.






    share|improve this answer











    $endgroup$









    • 1




      $begingroup$
      you are right! 740496902 is the result for 999
      $endgroup$
      – J42161217
      47 mins ago










    • $begingroup$
      Combining the first part of yours and second part of mine gives 25 bytes. Not sure which of us should have the combined version!
      $endgroup$
      – Nick Kennedy
      37 mins ago










    • $begingroup$
      @NickKennedy - nice, go for it!
      $endgroup$
      – Jonathan Allan
      35 mins ago
















    1












    $begingroup$


    Jelly, 30 bytes



    p`SÞ⁸ịð;Øp,²;¤×Ḟ¥/;+ƝQƊ⁹¡ị@ð/


    Try it online!

    This is a little slow, but a huge improvement is made with a prefix of Ḥ½Ċ (double, square-root, ceiling) like in this test-suite.






    share|improve this answer











    $endgroup$









    • 1




      $begingroup$
      you are right! 740496902 is the result for 999
      $endgroup$
      – J42161217
      47 mins ago










    • $begingroup$
      Combining the first part of yours and second part of mine gives 25 bytes. Not sure which of us should have the combined version!
      $endgroup$
      – Nick Kennedy
      37 mins ago










    • $begingroup$
      @NickKennedy - nice, go for it!
      $endgroup$
      – Jonathan Allan
      35 mins ago














    1












    1








    1





    $begingroup$


    Jelly, 30 bytes



    p`SÞ⁸ịð;Øp,²;¤×Ḟ¥/;+ƝQƊ⁹¡ị@ð/


    Try it online!

    This is a little slow, but a huge improvement is made with a prefix of Ḥ½Ċ (double, square-root, ceiling) like in this test-suite.






    share|improve this answer











    $endgroup$




    Jelly, 30 bytes



    p`SÞ⁸ịð;Øp,²;¤×Ḟ¥/;+ƝQƊ⁹¡ị@ð/


    Try it online!

    This is a little slow, but a huge improvement is made with a prefix of Ḥ½Ċ (double, square-root, ceiling) like in this test-suite.







    share|improve this answer














    share|improve this answer



    share|improve this answer








    edited 25 mins ago

























    answered 52 mins ago









    Jonathan AllanJonathan Allan

    54.3k537174




    54.3k537174








    • 1




      $begingroup$
      you are right! 740496902 is the result for 999
      $endgroup$
      – J42161217
      47 mins ago










    • $begingroup$
      Combining the first part of yours and second part of mine gives 25 bytes. Not sure which of us should have the combined version!
      $endgroup$
      – Nick Kennedy
      37 mins ago










    • $begingroup$
      @NickKennedy - nice, go for it!
      $endgroup$
      – Jonathan Allan
      35 mins ago














    • 1




      $begingroup$
      you are right! 740496902 is the result for 999
      $endgroup$
      – J42161217
      47 mins ago










    • $begingroup$
      Combining the first part of yours and second part of mine gives 25 bytes. Not sure which of us should have the combined version!
      $endgroup$
      – Nick Kennedy
      37 mins ago










    • $begingroup$
      @NickKennedy - nice, go for it!
      $endgroup$
      – Jonathan Allan
      35 mins ago








    1




    1




    $begingroup$
    you are right! 740496902 is the result for 999
    $endgroup$
    – J42161217
    47 mins ago




    $begingroup$
    you are right! 740496902 is the result for 999
    $endgroup$
    – J42161217
    47 mins ago












    $begingroup$
    Combining the first part of yours and second part of mine gives 25 bytes. Not sure which of us should have the combined version!
    $endgroup$
    – Nick Kennedy
    37 mins ago




    $begingroup$
    Combining the first part of yours and second part of mine gives 25 bytes. Not sure which of us should have the combined version!
    $endgroup$
    – Nick Kennedy
    37 mins ago












    $begingroup$
    @NickKennedy - nice, go for it!
    $endgroup$
    – Jonathan Allan
    35 mins ago




    $begingroup$
    @NickKennedy - nice, go for it!
    $endgroup$
    – Jonathan Allan
    35 mins ago











    1












    $begingroup$


    Jelly, 27 24 bytes



    p`SÞ⁸ịð’;×ØpḞ¥×⁹r‘ÆḞ¤Sð/


    Try it online!



    Monadic link using 1-based indexing.
    Thanks to @JonathanAllan for a better way of getting the row and columns from n and saving 3 bytes. In its shortest form it’s too slow for larger n on TIO, so the following Try it online! reduces the size of the initial list of rows and columns at the cost of three bytes.






    share|improve this answer











    $endgroup$













    • $begingroup$
      ...×⁹r‘ÆḞ¤Sð/ saves one in your amalgamation version (TIO)
      $endgroup$
      – Jonathan Allan
      9 mins ago


















    1












    $begingroup$


    Jelly, 27 24 bytes



    p`SÞ⁸ịð’;×ØpḞ¥×⁹r‘ÆḞ¤Sð/


    Try it online!



    Monadic link using 1-based indexing.
    Thanks to @JonathanAllan for a better way of getting the row and columns from n and saving 3 bytes. In its shortest form it’s too slow for larger n on TIO, so the following Try it online! reduces the size of the initial list of rows and columns at the cost of three bytes.






    share|improve this answer











    $endgroup$













    • $begingroup$
      ...×⁹r‘ÆḞ¤Sð/ saves one in your amalgamation version (TIO)
      $endgroup$
      – Jonathan Allan
      9 mins ago
















    1












    1








    1





    $begingroup$


    Jelly, 27 24 bytes



    p`SÞ⁸ịð’;×ØpḞ¥×⁹r‘ÆḞ¤Sð/


    Try it online!



    Monadic link using 1-based indexing.
    Thanks to @JonathanAllan for a better way of getting the row and columns from n and saving 3 bytes. In its shortest form it’s too slow for larger n on TIO, so the following Try it online! reduces the size of the initial list of rows and columns at the cost of three bytes.






    share|improve this answer











    $endgroup$




    Jelly, 27 24 bytes



    p`SÞ⁸ịð’;×ØpḞ¥×⁹r‘ÆḞ¤Sð/


    Try it online!



    Monadic link using 1-based indexing.
    Thanks to @JonathanAllan for a better way of getting the row and columns from n and saving 3 bytes. In its shortest form it’s too slow for larger n on TIO, so the following Try it online! reduces the size of the initial list of rows and columns at the cost of three bytes.







    share|improve this answer














    share|improve this answer



    share|improve this answer








    edited 2 mins ago

























    answered 48 mins ago









    Nick KennedyNick Kennedy

    1,56649




    1,56649












    • $begingroup$
      ...×⁹r‘ÆḞ¤Sð/ saves one in your amalgamation version (TIO)
      $endgroup$
      – Jonathan Allan
      9 mins ago




















    • $begingroup$
      ...×⁹r‘ÆḞ¤Sð/ saves one in your amalgamation version (TIO)
      $endgroup$
      – Jonathan Allan
      9 mins ago


















    $begingroup$
    ...×⁹r‘ÆḞ¤Sð/ saves one in your amalgamation version (TIO)
    $endgroup$
    – Jonathan Allan
    9 mins ago






    $begingroup$
    ...×⁹r‘ÆḞ¤Sð/ saves one in your amalgamation version (TIO)
    $endgroup$
    – Jonathan Allan
    9 mins ago













    0












    $begingroup$


    Wolfram Language (Mathematica), 90 bytes



    Flatten[Table[(F=Fibonacci)[a+1]⌊(b-a+1)GoldenRatio⌋+(b-a)F@a,{b,#},{a,b,1,-1}]][[#]]&


    Try it online!






    share|improve this answer









    $endgroup$


















      0












      $begingroup$


      Wolfram Language (Mathematica), 90 bytes



      Flatten[Table[(F=Fibonacci)[a+1]⌊(b-a+1)GoldenRatio⌋+(b-a)F@a,{b,#},{a,b,1,-1}]][[#]]&


      Try it online!






      share|improve this answer









      $endgroup$
















        0












        0








        0





        $begingroup$


        Wolfram Language (Mathematica), 90 bytes



        Flatten[Table[(F=Fibonacci)[a+1]⌊(b-a+1)GoldenRatio⌋+(b-a)F@a,{b,#},{a,b,1,-1}]][[#]]&


        Try it online!






        share|improve this answer









        $endgroup$




        Wolfram Language (Mathematica), 90 bytes



        Flatten[Table[(F=Fibonacci)[a+1]⌊(b-a+1)GoldenRatio⌋+(b-a)F@a,{b,#},{a,b,1,-1}]][[#]]&


        Try it online!







        share|improve this answer












        share|improve this answer



        share|improve this answer










        answered 1 hour ago









        J42161217J42161217

        14k21353




        14k21353






























            draft saved

            draft discarded




















































            If this is an answer to a challenge…




            • …Be sure to follow the challenge specification. However, please refrain from exploiting obvious loopholes. Answers abusing any of the standard loopholes are considered invalid. If you think a specification is unclear or underspecified, comment on the question instead.


            • …Try to optimize your score. For instance, answers to code-golf challenges should attempt to be as short as possible. You can always include a readable version of the code in addition to the competitive one.
              Explanations of your answer make it more interesting to read and are very much encouraged.


            • …Include a short header which indicates the language(s) of your code and its score, as defined by the challenge.



            More generally…




            • …Please make sure to answer the question and provide sufficient detail.


            • …Avoid asking for help, clarification or responding to other answers (use comments instead).





            draft saved


            draft discarded














            StackExchange.ready(
            function () {
            StackExchange.openid.initPostLogin('.new-post-login', 'https%3a%2f%2fcodegolf.stackexchange.com%2fquestions%2f183186%2fnew-order-5-where-fibonacci-and-beatty-meet-at-wythoff%23new-answer', 'question_page');
            }
            );

            Post as a guest















            Required, but never shown





















































            Required, but never shown














            Required, but never shown












            Required, but never shown







            Required, but never shown

































            Required, but never shown














            Required, but never shown












            Required, but never shown







            Required, but never shown







            Popular posts from this blog

            What are all the squawk codes?

            What are differences between VBoxVGA, VMSVGA and VBoxSVGA in VirtualBox?

            Olav Thon